You are on page 1of 41

[CHAPTER V: DEDUCTIONS] 1

G.R. Nos. L-28508-9 July 7, 1989


ESSO STANDARD EASTERN, INC., (formerly, Standard- HELD: No.
Vacuum Oil Company), petitioner,
vs.
THE COMMISSIONER OF INTERNAL Margin fee was imposed by the State in the exercise of its
REVENUE, respondent. police power and not the power of taxation. Hence, it is not
considered as tax.
TOPIC: Deductions
The applicable provision is Section 30(a) of the National
FACTS: Internal Revenue Code reading as follows:
SEC. 30. Deductions from gross income in
ESSO deducted from its gross income for 1959, as part of its computing net income there shall be allowed
ordinary and necessary business expenses, the amount it had as deductions
spent for drilling and exploration of its petroleum (a) Expenses:
concessions. The Commissioner disallowed the claim on the (1) In general. — All the ordinary and
ground that the expenses should be capitalized and might be necessary expenses paid or incurred during the
taxable year in carrying on any trade or
written off as a loss only when a “dry hole” should result.
business, including a reasonable allowance for
Hence, ESSO filed an amended return where it asked for the salaries or other compensation for personal
refund of P323,270 by reason of its abandonment, as dry services actually rendered; traveling expenses
holes, of several of its oil wells. It also claimed as ordinary while away from home in the pursuit of a trade
or business; and rentals or other payments
and necessary expenses in the same return amount
required to be made as a condition to the
representing margin fees it had paid to the Central Bank on continued use or possession, for the purpose of
its profit remittances to its New York Head Office. It the trade or business, of property to which the
contends that the margin fees are considered taxes and taxpayer has not taken or is not taking title or
in which he has no equity.
hence deductible. Alternatively, ESSO prays that if margin
fees are not taxes, they should nevertheless be considered (2) Expenses allowable to non-resident alien
individuals and foreign corporations. — In the
necessary and ordinary business expenses and therefore still case of a non-resident alien individual or a
deductible from its gross income. foreign corporation, the expenses deductible
are the necessary expenses paid or incurred in
ISSUE: Whether the margin fees may be considered ordinary carrying on any business or trade conducted
and necessary expenses when paid. within the Philippines exclusively.

Taxation 1- SY 2015-2016 (Zarate)


G.R. No.
cting claims, an expenses is considered necessary where the expenditure is appropriate and helpful L-15290
in the May
development of the31, 1963 business. It is ordinary when it connotes a
taxpayer’s
MARIANO ZAMORA, petitioner,
vs.
COLLECTOR OF INTERNAL REVENUE and COURT OF
TAX APPEALS, respondents.
TOPIC: Deductions
FACTS:

Mariano Zamora, owner of the Bay View Hotel and Farmacia


Zamora, Manila, filed his income tax returns the years 1951
and 1952. The Collector of Internal Revenue found that he
failed to file his return of the capital gains derived from the
sale of certain real properties and claimed deductions which
were not allowable. The collector required him to pay the
sums of P43,758.50 and P7,625.00, as deficiency income tax
for the years 1951 and 1952.

On appeal by Zamora, the Court of Tax Appeals modified the


decision appealed from and ordered him to pay the reduced
total sum of P30,258.00 (P22,980.00 and P7,278.00, as
deficiency income tax for the years 1951 and 1952.

Having failed to obtain a reconsideration of the decision,


Mariano Zamora appealed alleging that the Court of Tax
Appeals erred in disallowing P10,478.50, as promotion
expenses incurred by his wife for the promotion of the Bay
View Hotel and Farmacia Zamora (which is ½ of P20,957.00,
turn on mere equitable considerations.
2 CHAPTER V: DEDUCTIONS supposed business expenses). – eto lang related sa topic

Note: He contends that the whole amount of P20,957.00 as


[CHAPTER V: DEDUCTIONS] 3

promotion expenses in his 1951 income tax returns, should be the Bay View Hotel and Farmacia Zamora in the absence of
allowed and not merely one-half of it or P10,478.50, on the receipts proving the same.
ground that, while not all the itemized expenses are
supported by receipts, the absence of some supporting HELD: NO
receipts has been sufficiently and satisfactorily established.
For, as alleged, the said amount of P20,957.00 was spent by Section 30, of the Tax Code, provides that in computing net
Mrs. Esperanza A. Zamora (wife of Mariano), during her travel income, there shall be allowed as deductions all the ordinary
to Japan and the United States to purchase machinery for a and necessary expenses paid or incurred during the taxable
new Tiki-Tiki plant, and to observe hotel management in year, in carrying on any trade or business. Since promotion
modern hotels. The CTA, however, found that for said trip expenses constitute one of the deductions in conducting a
Mrs. Zamora obtained only the sum of P5,000.00 from the business, same must testify these requirements. Claim for
Central Bank and that in her application for dollar allocation, the deduction of promotion expenses or entertainment
she stated that she was going abroad on a combined medical expenses must also be substantiated or supported by record
and business trip, which facts were not denied by Mariano showing in detail the amount and nature of the expenses
Zamora. No evidence had been submitted as to where incurred (N.H. Van Socklan, Jr. v. Comm. of Int. Rev.; 33 BTA
Mariano had obtained the amount in excess of P5,000.00 544). Considering, as heretofore stated, that the application
given to his wife which she spent abroad. No explanation had of Mrs. Zamora for dollar allocation shows that she went
been made either that the statement contained in Mrs. abroad on a combined medical and business trip, not all of
Zamora's application for dollar allocation that she was going her expenses came under the category of ordinary and
abroad on a combined medical and business trip, was not necessary expenses; part thereof constituted her personal
correct. The alleged expenses were not supported by expenses. There having been no means by which to ascertain
receipts. Mrs. Zamora could not even remember how much which expense was incurred by her in connection with the
money she had when she left abroad in 1951, and how the business of Mariano Zamora and which was incurred for her
alleged amount of P20,957.00 was spent. personal benefit, the Collector and the CTA in their
decisions, considered 50% of the said amount of P20,957.00
ISSUE: as business expenses and the other 50%, as her personal
Whether or not the CTA erred in disallowing P10,478.50 as expenses. We hold that said allocation is very fair to Mariano
promotion expenses incurred by his wife for the promotion of Zamora, there having been no receipt whatsoever, submitted
to explain the alleged business expenses, or proof of the

Taxation 1- SY 2015-2016 (Zarate)


connection which said expenses had to the business or the
reasonableness of the said amount of P20,957.00. While in
situations like the present, absolute certainty is usually not
possible, the CTA should make as close an approximation as it
can, bearing heavily, if it chooses, upon the taxpayer whose
inexactness is of his own making. KUENZLE & STREIFF INC. V CIR
Facts:
In the case of Visayan Cebu Terminal Co., Inc. v. Collector of Kuenzle & Streiff for the years 1953, 1954 and 1955 filed its
Int. Rev, it was declared that representation expenses fall income tax return, declaring losses.
under the category of business expenses which are allowable CIR filed for deficiency of income taxes against Kuenzle & Streiff
Inc. for the said years in the amounts of P40, 455.00, P11, 248.00
deductions from gross income, if they meet the conditions
and P16, 228.00, respectively, arising from the disallowance, as
prescribed by law, particularly section 30 (a) [1], of the Tax deductible expenses, of the bonuses paid by the corporation to its
Code; that to be deductible, said business expenses must be officers, upon the ground that they were not ordinary, nor
ordinary and necessary expenses paid or incurred in carrying necessary, nor reasonable expenses within the purview of Section
on any trade or business; that those expenses must also meet 30(a) (1) of the National Internal Revenue Code.
the further test of reasonableness in amount; that when The corporation filed with the Court of Tax Appeals a petition for
some of the representation expenses claimed by the taxpayer review contesting the assessments. CTA favored the CIR, however
were evidenced by vouchers or chits, but others were lowered the tax due on 1954. The corporation moved for
without vouchers or chits, documents or supporting papers; reconsideration, but still lost.
that there is no more than oral proof to the effect that
The Corporation contends that the tax court, in arriving at its
payments have been made for representation expenses conclusion, acted "in a purely arbitrary manner", and erred in not
allegedly made by the taxpayer and about the general nature considering individually the total compensation paid to each of
of such alleged expenses; that accordingly, it is not possible petitioner's officers and staff members in determining the
to determine the actual amount covered by supporting reasonableness of the bonuses in question, and that it erred
likewise in holding that there was nothing in the record indicating
papers and the amount without supporting papers, the court
that the actuation of the respondent was unreasonable or unjust.
should determine from all available data, the amount ISSUE: Whether or not the bonuses in question was reasonable
4 CHAPTER
properly V: DEDUCTIONS
deductible as representation expenses. and just to be allowed as a deduction?
HELD: No.
RATIO: It is a general rule that `Bonuses to employees made in
[CHAPTER V: DEDUCTIONS] 5

good faith and as additional compensation for the services that contributed materially to the success of petitioner's business
actually rendered by the employees are deductible, provided during the taxable years in question.
such payments, when added to the stipulated salaries, do not 2) All the other employees received no pay increase in the said
exceed a reasonable compensation for the services rendered. years.
The condition precedents to the deduction of bonuses to employees 3) The bonuses were paid despite the fact that it had suffered net
are: (1) the payment of the bonuses is in fact compensation; (2) it losses for 3 years. Furthermore the corporation cannot use the
must be for personal services actually rendered; and (3) bonuses, excuse that it is 'salary paid' to an employee because the CIR does
when added to the salaries, are `reasonable ... when measured by not question the basic salaries paid by petitioner to the officers
the amount and quality of the services performed with relation to and employees, but disallowed only the bonuses paid to petitioner's
the business of the particular taxpayer. Here it is admitted that the top officers at the end of the taxable years in question.
bonuses are in fact compensation and were paid for services
actually rendered. The only question is whether the payment of ALTERNATIVE DIGEST FOR
said bonuses is reasonable.
G.R. No. L-18840 May 29, 1969
There is no fixed test for determining the reasonableness of a given KUENZLE & STREIFF, INC., petitioner,
bonus as compensation. This depends upon many factors, one of vs.
them being the amount and quality of the services performed with THE COMMISSIONER OF INTERNAL
relation to the business. Other tests suggested are: payment must REVENUE, respondent.
be 'made in good faith'; the character of the taxpayer's business,
the volume and amount of its net earnings, its locality, the type Topic: Deductions
and extent of the services rendered, the salary policy of the
corporation'; 'the size of the particular business'; 'the employees'
qualifications and contributions to the business venture'; and FACTS:
'general economic conditions. However, 'in determining whether the
particular salary or compensation payment is reasonable, the Kuenzle & Streiff for the years 1953, 1954 and 1955 filed its
situation must be considered as a whole. income tax return, declaring losses.
It seems clear from the record that, in arriving at its main
conclusion, the tax court considered, inter alia, the following CIR filed for deficiency of income taxes against Kuenzle &
factors:
1) The paid officers, in the absence of evidence to the contrary,
Streiff Inc. for the said years in the amounts of P40,455.00,
that they were competent, on the other the record discloses no P11,248.00 and P16,228.00, respectively, arising from the
evidence nor has petitioner ever made the claim that all or some of disallowance, as deductible expenses, of the bonuses paid by
them were gifted with some special talent, or had undergone some the corporation to its officers, upon the ground that they
extraordinary training, or had accomplished any particular task, were not ordinary, nor necessary, nor reasonable expenses

Taxation 1- SY 2015-2016 (Zarate)


within the purview of Section 30(a) (1) of the National rendered; and (3) bonuses, when added to the salaries, are
Internal Revenue Code. `reasonable ... when measured by the amount and quality of
the services performed with relation to the business of the
The corporation filed with the Court of Tax Appeals a petition particular taxpayer. Here it is admitted that the bonuses are
for review contesting the assessments. CTA favored the CIR, in fact compensation and were paid for services actually
however lowered the tax due on 1954. The corporation rendered. The only question is whether the payment of said
moved for reconsideration, but still lost. bonuses is reasonable.

The Corporation contends that the tax court, in arriving at its There is no fixed test for determining the reasonableness of a
conclusion, acted "in a purely arbitrary manner", and erred in given bonus as compensation. This depends upon many
not considering individually the total compensation paid to factors, one of them being the amount and quality of the
each of petitioner's officers and staff members in services performed with relation to the business. Other tests
determining the reasonableness of the bonuses in question, suggested are: payment must be 'made in good faith'; the
and that it erred likewise in holding that there was nothing in character of the taxpayer's business, the volume and amount
the record indicating that the actuation of the respondent of its net earnings, its locality, the type and extent of the
was unreasonable or unjust. services rendered, the salary policy of the corporation'; 'the
size of the particular business'; 'the employees' qualifications
ISSUE: Whether or not the bonuses in question was
reasonable and just to be allowed as a deduction? and contributions to the business venture'; and 'general
economic conditions. However, 'in determining whether the
HELD: No. particular salary or compensation payment is reasonable, the
situation must be considered as a whole.
It is a general rule that `Bonuses to employees made in good
faith and as additional compensation for the services actually It seems clear from the record that, in arriving at its main
rendered by the employees are deductible, provided such conclusion, the tax court considered, inter alia, the
payments, when added to the stipulated salaries, do not following factors:
exceed a reasonable compensation for the services rendered. 1) The paid officers, in the absence of evidence to the
The6condition
CHAPTER V: DEDUCTIONS
precedents to the deduction of bonuses to contrary, that they were competent, on the other the record
employees are: (1) the payment of the bonuses is in fact discloses no evidence nor has petitioner ever made the
compensation; (2) it must be for personal services actually claim that all or some of them were gifted with some special
[CHAPTER V: DEDUCTIONS] 7

talent, or had undergone some extraordinary training, or had considered as ordinary or necessary — and was therefore
accomplished any particular task, that contributed materially beyond the purview of the provisions of Section 30(a) (1) of
to the success of petitioner's business during the taxable the National Internal Revenue Code. This being so, We cannot
years in question. see our way clear to holding that the respondent acted
arbitrarily in disallowing as deductible expenses the amounts
2) All the other employees received no pay increase in thus paid as bonus or "additional remuneration".
the said years.

3) The above salaries and bonuses were paid to petitioner's C. M. Hoskins & Co. Inc. v Commissioner of Internal Revenue
top officials mentioned heretofore, in spite of the fact that Facts: Petitioner, a domestic corporation engaged in the real
according to its income tax returns for the relevant years, it estate business as brokers, managing agents and
had suffered net losses. In fact, petitioner's financial administrators, filed its income tax return for its fiscal year
statements further show that its gross assets suffered a ending September 30, 1957 showing a net income of
gradual decrease for the same years.), and that a similar P92,540.25 and a tax liability due thereon of P18,508.00,
downward trend took place in its surplus and capital position which it paid in due course. Upon verification of its return,
during the same period of time. respondent Commissioner of Internal Revenue, disallowed
four items of deduction in petitioner's tax returns and
Petitioner admits that the amounts it paid to its top officers assessed against it an income tax deficiency in the amount of
in 1953 as bonus or "additional remuneration" were taken P28,054.00 plus interests. The Court of Tax Appeals upon
either from operating funds, that is, funds from the year's reviewing the assessment at the taxpayer's petition, upheld
respondent's disallowance of the principal item of petitioner's
business operations, or from its general reserve. Normally,
having paid to Mr. C. M. Hoskins, its founder and controlling
the amounts taken from the first source should have
stockholder the amount of P99,977.91 representing 50% of
constituted profits of the corporation distributable as
supervision fees earned by it and set aside respondent's
dividends amongst its shareholders. Instead it would appear disallowance of three other minor items. The Tax Court
that they were diverted from this purpose and used to pay therefore determined petitioner's tax deficiency to be in the
the bonuses for the year 1953. In the case of the amounts amount of P27,145.00 and on November 8, 1964 rendered
taken from the general reserve it seems clear that the judgment against it, as follows:
company had to resort to the use of such reserve funds
because the item of expense to be met could not be

Taxation 1- SY 2015-2016 (Zarate)


WHEREFORE, premises considered, the decision of the · Must be for personal services actually rendered
respondent is hereby modified. Petitioner is ordered to pay
to the latter or his representative the sum of P27,145.00, · Bonuses when added to salaries are reasonable when
representing deficiency income tax for the year 1957, plus measured by the amount and quality of services performed
interest at 1/2% per month from June 20, 1959 to be with relation to the business of the particular taxpayer.
computed in accordance with the provisions of Section 51(d)
of the National Internal Revenue Code. If the deficiency tax There is no fixed test for determining the reasonableness of a
is not paid within thirty (30) days from the date this decision given bonus as compensation. This depends upon many
becomes final, petitioner is also ordered to pay surcharge factors.
and interest as provided for in Section 51 (e) of the Tax Code,
without costs.
In the case, Hoskins fails to pass the test. CTA was correct in
Issue:
holding that the payment of the company to Mr. Hoskins of
Whether or not the disallowance of the 4 items were the sum P99,977.91 as 50% share of supervision fees received
proper. by the company was inordinately large and could not be
treated as an ordinary and necessary expenses allowed for
Held: deduction.

NOT deductible. It did not pass the test of reasonableness It is a general rule that `Bonuses to employees made in good
which is: faith and as additional compensation for the services actually
rendered by the employees are deductible, provided such
General rule, bonuses to employees made in good faith and
payments, when added to the stipulated salaries, do not exceed a
as additional compensation for services actually rendered by
the employees are deductible, provided such payments, reasonable compensation for the services rendered.
when added to the salaries do not exceed the compensation The condition precedents to the deduction of bonuses to
for services rendered. employees are: (1) the payment of the bonuses is in fact
compensation; (2) it must be for personal services actually
The conditions precedent to the deduction of bonuses to rendered; and (3) bonuses, when added to the salaries, are
emp loy e e s a re : `reasonable ... when measured by the amount and quality of the
8 C H A P T E R V: DEDUCTIONS
services performed with relation to the business of the particular
· Payment of bonuses is in fact compensation
taxpayer. Here it is admitted that the
[CHAPTER V: DEDUCTIONS] 9

bonuses are in fact compensation and were paid for CA however, set aside the ruling of the CTA: “Since it has
services actually rendered. not been sufficiently established that the item it claimed as
a deduction is excessive, the same should be allowed.”

Thus, this petition by CIR.


CIR vs. GENERAL FOODS PHILS. INC. (test of reasonableness)
ISSUE: W/N the media advertising expense for Tang incurred
FACTS: In June 1985, GF, engaged in the manufacture of
by respondent corporation was an ordinary and necessary
beverages filed its income tax return for the fiscal year
expense fully deductible under the
ending February 28, 1985. In said tax return, respondent
corporation claimed as deduction, among other business HELD: NO. We find the subject expense for the
expenses, the amount of P9,461,246 for media advertising for advertisement of a single product to be inordinately large.
Tang. Therefore, even if it is necessary, it cannot be considered an
ordinary expense deductible under the NIRC.
CIR disallowed 50% or P4,730,623 of the deduction claimed
by respondent corporation. Consequently, GR was assessed Advertising is generally of two kinds: (1) advertising to
deficiency income taxes in the amount of P2,635, stimulate the current sale of merchandise or use of services
141.42. The latter filed a motion for reconsideration but the and (2) advertising designed to stimulate the future sale of
same was denied. merchandise or use of services. The second type involves
expenditures incurred, in whole or in part, to create or
GR appealed to the Court of Tax Appeals but the appeal was
maintain some form of goodwill for the taxpayers trade or
dismissed: “With such a gargantuan expense for the
business or for the industry or profession of which the
advertisement of a singular product, which even excludes
taxpayer is a member. If the expenditures are for the
other advertising and promotions expenses, we are not
advertising of the first kind, then, except as to the question
prepared to accept that such amount is reasonable to
of the reasonableness of amount, there is no doubt such
stimulate the current sale of merchandise regardless of
expenditures are deductible as business expenses. If,
Petitioners explanation that such expense does not connote
however, the expenditures are for advertising of the second
unreasonableness considering the grave economic situation
kind, then normally they should be spread out over a
taking place after the Aquino assassination characterized by
reasonable period of time.
capital fight, strong deterioration of the purchasing power
of the Philippine peso and the slacking demand for We agree with the CTA that the subject advertising expense
consumer products”. was of the second kind. Not only was the amount staggering;

Taxation 1- SY 2015-2016 (Zarate)


the respondent corporation itself also admitted, in its letter professional and security services were properly claimed by
protest to the Commissioner of Internal Revenues ICC in 1986 because it was only in the said year when the
assessment, that the subject media expense was incurred in bills demanding payment were sent to ICC. Hence, even if
order to protect GR’s brand franchise, a critical point during some of these professional services were rendered to ICC in
the period under review. 1984 or 1985, it could not declare the same as deduction for
The protection of brand franchise is analogous to the the said years as the amount thereof could not be
maintenance of goodwill or title to one’s property. This is a determined at that time.
capital expenditure which should be spread out over a CIR appealed to the CA which upheld CTA holding that
reasonable period of time. although the professional services (legal and auditing
Respondent corporations venture to protect its brand services) were rendered to ICC in 1984 and 1985, the cost of
franchise was tantamount to efforts to establish a the services was not yet determinable at that time, hence, it
reputation. This was akin to the acquisition of capital assets could be considered as deductible expenses only in 1986
and therefore expenses related thereto were not to be when ICC received the billing statements for said services.
considered as business expenses but as capital expenditures. Hence, this case before the SC.

ISSUE: W/N the deduction of the expenses for professional


CIR vs. ISABELA CULTURAL CORPORATION (it must be paid and security services from ICCs gross income was correct.
or incurred during the taxable year) HELD: NO.
FACTS: In February 1990, ICC, a domestic corporation, The requisites for the deductibility of ordinary and necessary
received from the BIR an Assessment Notice for deficiency trade, business, or professional expenses, like expenses paid
income tax in the amount of P333,196.86, and an Assessment for legal and auditing services, are: (a) the expense must be
Notice for deficiency expanded withholding tax in the ordinary and necessary; (b) it must have been paid or
amount of P4,897.79, inclusive of surcharges and interest, incurred during the taxable year; (c) it must have been
both for the taxable year 1986. paid or incurred in carrying on the trade or business of the
taxpayer; and (d) it must be supported by receipts, records
Th1e0deCfiHcAiePnTcyERexVp:anDdEeDdUwCitThIhOoNldSing tax was allegedly due
or other pertinent papers.
ure of ICC to withhold 1% expanded withholding tax on its claimed P244,890.00 deduction for security services.
eld however that the claimed deductions for The requisite that it must have been paid or incurred during
the taxable year is further qualified by Section 45 of the
[CHAPTER V: DEDUCTIONS] 11

National Internal Revenue Code (NIRC) which states that: Philippines. It is engaged in the importation, manufacture,
[t]he deduction provided for in this Title shall be taken for and sale of pharmaceutical drugs and chemicals.
the taxable year in which paid or accrued or paid or
incurred, dependent upon the method of accounting upon In 1971, it declared a net taxable income of P1.4 M and paid
the basis of which the net income is computed P511k as tax due. It claimed its share of the head office
overhead expenses (P501k) as deduction from gross income.
From the nature of the claimed deductions and the span of In its amended return, it claimed that there was an
time during which the firm was retained, ICC can be overpayment of tax (P324k) arising from under-deduction of
expected to have reasonably known the retainer fees charged the overhead expense. This was certified by international
by the firm as well as the compensation for its legal independent auditors, the allocation of the overhead expense
services. The failure to determine the exact amount of the made on the basis of the percentage of gross income in the
expense during the taxable year when they could have been Philippines to gross income of the corporation as a whole.
claimed as deductions cannot thus be attributed solely to the
delayed billing of these liabilities by the firm. For one, ICC, In 1974, without waiting for the action of the CIR, Smith filed
in the exercise of due diligence could have inquired into the a petition for review with the CTA. CTA ordered CIR to
amount of their obligation to the firm, especially so that it is refund the overpayment or grant Smith a tax credit. CIR
using the accrual method of accounting. For another, it could appealed to the SC.
have reasonably determined the amount of legal and retainer
fees owing to its familiarity with the rates charged by their Issue: Whether Smith is entitled to a refund – YES
long time legal consultant.
Ratio:

CIR v CTA AND SMITH&FRENCH OVERSEAS


The governing law is found in Sec. 37 (b).1 Revenue

Facts: Regulation No. 2 of the DOF contains a similar provision, with


the additional line that “the ratable part is based upon the
Smith Kline & French Overseas Company is a multinational ratio of gross income from sources within the Philippines to
firm domiciled in Philadelphia, licensed to do business in the the total gross income” (Sec. 160). Hence, where an
expense is clearly related to the production of Philippine-

1
Net income from sources in the Philippines. – From the items of gross income specified in subsection (a) of this section there shall be
deducted expenses, losses, and other deductions properly apportioned or allocated thereto and a ratable part of any expenses, losses, or other
deductions which cannot definitely be allocated to some item or class of gross income. The remainder, if any, shall be included in full as net
income from sources within the Philippines.

Taxation 1- SY 2015-2016 (Zarate)


derived income or to Philippine operations, that expense can between private parties nor can the CIR acquiesce in such an
be deducted from the gross income acquired in the agreement.
Philippines without resorting to apportionment.
SC ruled for Smith Kline and said that its amended return
However, the overhead expenses incurred by the parent conforms with the law and regulations.
company in connection with finance, administration, and
research & development, all of which directly benefit its
branches all over the world, fall under a different category. GUTIERREZ v COLLECTOR
These are items which cannot be definitely allocated or
identified with the operations of the Philippine branch. Facts:
Smith can claim as its deductible share a ratable part of such Lino Gutierrez was primarily engaged in the business of
expenses based upon the ration of the local branch’s gross leasing real property for which he paid real estate broker’s
privilege tax. The Collector assessed against Gutierrez
income to the total gross income of the corporation deficiency income tax amounting to P11,841.
worldwide.
The deficiency tax came about by the disallowance of
CIR’s Contention deductions from gross income representing depreciation
expenses Gutierrez allegedly incurred in carrying on his
The CIR does not dispute the right of Smith to avail of Sec. 37 business. The expenses consisted of:
(b) of the Tax Code and Sec. 160 of the RR. But he maintains
that such right is not absolute and that there exists a 1. Transportation expenses incurred to attend the
contract (service agreement) which Smith has entered into funeral of his friends,
with its home office, prescribing the amount that a branch 2. Procurement and installation of an iron door,
3. Cost of furniture given by the taxpayer in furtherance
can deduct as its share of the main office’s overhead
of a business transaction,
expenses. Since the share of the Philippine branch has been 4. Membership fees in organizations established by those
fixed, Smith cannot claim more than the said amount. engaged in the real estate trade,
5. Car expenses, salary of his driver and car depreciation,
Smith’s Contention 6. Repairing taxpayer’s rental apartments,
7. Litigation expenses,
Smith, on the other hand, submits that the contract between 8. Depreciation of Gutierrez’ residence,
its1e2lf CaHndAPiTtsERhoVm: eDEoDfUfiCceTIOcaNnSnot 9. Fines and penalties for late payment of taxes,
amend tax laws and regulations. The matter of allocated 10. Alms given to in indigent family and a donation
expenses deductible under the law cannot be the subject of consisting of officer’s jewels and aprons to Biak-na-Bato
an agreement Lodge No. 7.
[CHAPTER V: DEDUCTIONS] 13

established by those engaged in the real estate trade. Having


Issue: proved that his, the expenses incurred are deductible as
Whether or not claims for deduction are proper and ordinary and necessary business expenses.
allowable.
5. Car expenses, salary of his driver and car depreciation –
Held: 1/3 of the same was disallowed by the Commissioner on the
To be deductible, an expense must be: ground that the taxpayer used his car and driver both for
· Ordinary and necessary personal and business purposes. There is no clear showing,
· Paid or incurred within the taxable year however, that the car was devoted more for the taxpayer's
· Paid or incurred in carrying on a trade or business. business than for his personal and business needs.
According to the evidence, the taxpayer's car was utilized
1. Transportation expenses which petitioner incurred to both for personal and business needs. It is reasonable to
attend the funeral of his friends and the cost of admission allow as deduction 1/2 of the driver's salary, car expenses
tickets to operas - expenses relative to his personal and and depreciation.
social activities rather than to his business of leasing real
estate. 6. Those used to repair the taxpayer's rental apartments - did
not increase the value of such apartments, or prolong their
2. Procurement and installation of an iron door to - purely a life. They merely kept the apartments in an ordinary
personal expense. Personal, living, or family expenses are not operating condition. Hence, the expenses incurred are
deductible. deductible as necessary expenditures for the maintenance of
the taxpayer's business.
3. Cost of furniture given by the taxpayer as commission in 7. Litigation expenses - defrayed by Gutierrez to collect
furtherance of a business transaction - the expenses incurred apartment rentals and to eject delinquent tenants are
in attending the National Convention of Filipino Businessmen, ordinary and necessary expenses in pursuing his business. It is
luncheon meeting and cruise to Corregidor of the routinary and necessary for one in the leasing business to
Homeowners' Association were shown to have been made in collect rentals and to eject tenants who refuse to pay their
the pursuit of his business. Commissions given in accounts.
consideration for bringing about a profitable transaction are
part of the cost of the business transaction and are 8. Depreciation of Gutierrez' residence - not deductible. A
deductible. taxpayer may deduct from gross income a reasonable
allowance for deterioration of property arising out of its use
4. Membership and activities in connection therewith were or employment in business or trade. Gutierrez' residence was
solely to enhance his business -Gutierrez was an officer of not used in his trade or business.
the Junior Chamber of Commerce which sponsored the
National Convention of Filipino Businessmen. He was also the 9. Deduction the fines and penalties which he paid for late
president of the Homeowners' Association, an organization payment of taxes - while Section 30 allows taxes to be

Taxation 1- SY 2015-2016 (Zarate)


deducted from gross income, it does not specifically allow
informed petitioner that his income tax defendant
fines and penalties to be so deducted.
efficiency for 1949 amounted to P16,860.31.
▪ On April 15, 1956, respondent issued a warrant of distraint
Deductions from gross income are matters of legislative
and levy against the properties of Gancayco for the
grace; what is not expressly granted by Congress is withheld.
satisfaction of his deficiency income tax liability, and
Moreover, when acts are condemned, by law and their accordingly, the municipal treasurer of Catanauan, Quezon
commission is made punishable by fines or forfeitures, to issued on May 29, 1956, a notice of sale of said property at
allow them to be deducted from the wrongdoer's gross public auction on June 19, 1956.
income, reduces, and so in part defeats, the prescribed
▪ The question whether the sum of P16,860.31 is due from
punishment. Gancayco as deficiency income tax for 1949 hinges on the
validity of his claim for deduction of two (2) items, namely:
10. Alms to an indigent family and various individuals, (a) for farming expenses, P27,459.00; and (b) for
contributions to Lydia Yamson and G. Trinidad and a donation representation expenses, P8,933.45.
consisting of officers' jewels and aprons to Biak-na-Bato
Lodge No. 7 - not deductible from gross income inasmuch as ISSUE:
their recipients have not been shown to be among those
specified by law. Contributions are deductible when given to
Whether or not the 2 claimed deductions are allowable.
the Government of the Philippines, or any of its political
subdivisions for exclusively public purposes, to domestic
corporations or associations organized and operated
HELD:
exclusively for religious, charitable, scientific, athletic,
cultural or educational purposes, or for the rehabilitation of
veterans, or to societies for the prevention of cruelty to No.
children or animals, no part of the net income of which
inures to the benefit of any private stockholder or individual. Section 30 of the Tax Code partly reads: (a) Expenses: (1) In
General — All the ordinary and necessary expenses paid or
incurred during the taxable year incarrying on any trade or
business, including a reasonable allowance for salaries or
GANGAYCO vs COLLECTOR other compensation for personal services actually rendered;
traveling expenses while away from home in the pursuit of a
FACTS: trade or business; and rentals or other payments required to
▪ Respondent CIR issued a communication informing the be made as a condition to the continued use or possession,
petitioner thatV:there was still due from him, a efficiency for the purposes of the trade or business, of property to
14 CHAPTER DEDUCTIONS which the taxpayer has not taken or is not taking title or in
income tax for the year 1949, the sum of P29,554.05.
Gancayco sought a reconsideration, which was part granted which he has no equity.
by respondent, who in a letter dated April 8, 1953,
[CHAPTER V: DEDUCTIONS] 15

In computing net income no deduction shall in any case be


manufacturer, wholesaler, and distributor in the Philippines
allowed in respect of —(1) Personal, living, or family
of all products of 3M-St. Paul. To enable it to manufacture,
expenses;(2) Any amount paid out for new buildings or
package, promote, market, sell and install the highly
for permanent improvements, or betterments made to
specialized products of its parent company, and render the
increase the value of any property or estate;(3) Any amount
necessary post-sales service and maintenance to its
expended in restoring property or in making good the
customers, petitioner entered into a "Service Information
exhaustion thereof for which an allowance is or has been
and Technical Assistance Agreement" and a "Patent and
made; or (4) Premiums paid on any life insurance policy
Trademark License Agreement" with the latter under which
covering the life of any officer or employee, or any person
the petitioner agreed to pay to 3M-St. Paul a technical
financially interested in any trade or business carried on by
service fee of 3% and a royalty of 2% of its net sales.
the taxpayer, individual or corporate, when the taxpayer is
▪ In its income tax return for the fiscal year ended October
directly or indirectly a beneficiary under such policy.
31, 1974, the petitioner claimed the following deductions
as business expenses:(a) royalties and technical service fees
The cost of farm machinery, equipment and farm building
of P 3,050,646.00; and (b) pre-operational cost of tape
represents a capital investment and is not an allowable coater of P97,485.08.
deduction as an item of expense. Amounts expended in The Commissioner did not allow the entire amount as

the development of farms, orchards, and ranches prior to
deduction. Instead, respondent ordered petitioner to pay
the time when the productive state is reached may be P840,540 as deficiency income tax on its 1974 return, plus
regarded as investments of capital. Accordingly, they are not P353,026.80 as 14% interest per annum from February 15,
deductible. As for Gangayco’s claim for representation 1975 to February 15, 1976, or a total ofP1,193,566.80.
expenses amounting to P31,753.97, of which P22,820.52 was Petitioner protested and argued that the law applicable to

allowed, and P8,933.45 disallowed. Such disallowance is its case is only Section 29(a)(1) and not circular 393 of the
justified by the record, for, apart from the absence of central bank.
receipts, invoices or vouchers of the expenditures in CA upheld the commissioner’s ruling.

question, petitioner could not specify the items constituting
the same, or when or on whom or on what they were
incurred. ISSUE:
Whether or not the royalty payments are valid deductible
payments.
3M Philippines vs. CIR
FACTS:
HELD:
▪ 3M Philippines, Inc. is a subsidiary of the Minnesota Mining
and Manufacturing Company (or "3M-St. Paul") a non- No.
resident foreign corporation with principal office in St.
Paul, Minnesota, U.S.A. It is the exclusive importer,

Taxation 1- SY 2015-2016 (Zarate)


Although although the Tax Code allows payments of royalty to
In 1956, respondent filed with BIR his income tax return for
be deducted from gross income as business expenses, it is CB
Circular No. 393 that defines what royalty payments are the calendar year 1955, claiming, among others, a deduction
proper. Hence, improper payments of royalty are not for interest. On the basis of this return, he was assessed the
deductible as legitimate business expenses. Section 3c of the sum of P21,052.91, as income tax, which he paid.
circular provides that no royalty is payable on the wholesale Subsequently, the respondent filed an amended return for
price of finished products imported by the licensee from the the year 1955, claiming an additional deduction representing
licensor. Further, Circulars issued by the Central Bank in the interest paid on the donee's gift tax. The claim for deduction
exercise of its authority under the Central Bank Act, and
which have been duly published in the Official Gazette, have was based on the provisions of Section 30(b) (1) of the Tax
the force and effect of law (People vs. Que Po Lay, 94 Phil. Code, which authorizes the deduction from gross income of
640; Lim Hoa Ting vs. Central Bank, 104 Phil. 573). They are interest paid within the taxable year on indebtedness. A
binding on everybody, the petitioner, as much as the public claim for the refund of alleged overpaid income taxes for the
respondent. year 1955 amounting to P17,885.01, which is the difference
between the amount of P21,052.01 he paid as income taxes
under his original return and of P3,167.00, was filed together
with this amended return. The claim for refund was denied
OPTIONAL TREATMENT OF INTEREST EXPENSE
by the BIR.
COMMISSIONER OF INTERNAL REVENUE vs. CARLOS
PALANCA, JR. (1966)
In 1957, the respondent reiterated his claim for refund but
was again denied. Then, he requested the case to be referred
In 1950, Don Palanca Sr. donated in favor of his son to the conference staff of the BIR. Later, he requested the
(Respondent) shares of stock in La Tondeña, Inc. amounting respondent to hold his action on the case in abeyance until
to 12,500 shares. Respondent failed to file a return on the after the CTA renders its division on a similar case, and on
donation within the statutory period so BIR assessed the sums November 7, 1957, BIR denied the claim for the refund.
of P97,691.23, P24,442.81 and P47,868.70 as gift tax, 25%
surcharge and interest, respectively, which he paid on June
22, 1955.
Meanwhile, the BIR considered the transfer of 12,500 shares
16 CHAPTER V: DEDUCTIONS
of stock of La Tondeña Inc. to be a transfer in contemplation
of death pursuant to Section 88(b) of the NIRC. Consequently,
the respondent assessed against the petitioner the sum of
[CHAPTER V: DEDUCTIONS] 17

P191,591.62 as estate and inheritance taxes on the transfer ISSUE: WON the amount paid by Palanca for interest on his
of said 12,500 shares of stock. The amount of P17,002.74 delinquent estate and inheritance tax is deductible from the
paid on June 22, 1955 by the petitioner as gift tax, including gross income for that year under Section 30 (b) (1) of the
interest and surcharge, was applied to his estate and Revenue Code.
inheritance tax liability. On the tax liability of P191,591.62,
the petitioner paid the amount of P60,581.80 as interest for
delinquency.
HELD: Yes.

While "taxes" and "debts" are distinguishable legal concepts,


In 1958, the respondent once more filed an amended income in certain cases as in the suit at bar, on account of their
tax return for the year 1955, claiming, in addition to the nature, the distinction becomes inconsequential. This
interest deduction of P9,076.45 appearing in his original qualification is recognized even in the United States. Thus,
return, a deduction in the amount of P60,581.80, The term "debt" is properly used in a comprehensive sense as
representing interest on the estate and inheritance taxes on embracing not merely money due by contract, but whatever
the 12,500 shares of stock, thereby reporting a net taxable one is bound to render to another, either for contract or the
income for 1955 in the amount of P5,400.32 and an income requirements of the law. (Camden vs. Fink Coule and Coke
tax due thereon in the sum of P428.00. He attached a letter Co., 61 ALR 584).
requesting the refund of P20,624.01 which is the difference
between the amounts of P21,052.01 he paid as income tax Where statutes impose a personal liability for a tax, the tax
under his original return and of P428.00. Without waiting for becomes at least in a broad sense, a debt. (Idem.) Some
the CIR's decision on this claim for refund, he filed his American authorities hold that, especially for remedial
petition for review before the CTA. CIR then denied the purposes, Federal taxes are debts. (Tax Commission vs.
request for refund. National Malleable Castings Co., 35 ALR 1448)

CTA ordered CIR to refund to the respondent the amount


representing alleged over-payment of income taxes for the
In our jurisdiction, the rule is settled that although taxes
calendar year 1955.
already due have not, strictly speaking, the same concept as
debts, they are, however obligations that may be considered
as such. (Sambrano vs. Court of Tax Appeals, G.R. no. L-8652,
March 30, 1957). In a more recent case Commissioner of
Internal Revenue vs. Prieto, G.R. No. L-13912, September 30,

Taxation 1- SY 2015-2016 (Zarate)


1960, we explicitly announced that while the distinction preferred non-pioneer enterprise with respect to its
between "taxes" and "debts" was recognized in this integrated plywood and veneer mills.
jurisdiction, the variance in their legal conception does not
extend to the interests paid on them, at least insofar as In 1983, Picop received from the CIR 2 letters of assessment
Section 30 (b) (1) of the National Internal Revenue Code is and demand: (a) one for deficiency transaction tax and for
concerned. documentary and science stamp tax; and (b) the other for
deficiency income tax for 1977, for an aggregate amount of
P88,763,255.00. Picop protested the assessment of deficiency
We do not see any element in this case which can justify a transaction tax and documentary and science stamp taxes. It
departure from or abandonment of the doctrine in the Prieto also protested the deficiency income tax assessment for
case above. In both this and the said case, the taxpayer 1977. These protests were not formally acted upon by CIR. In
sought the allowance as deductible items from the gross 1984, the CIR issued a warrant of distraint on personal
income of the amounts paid by them as interests on property and a warrant of levy on real property against
delinquent tax liabilities. Of course, what was involved in the Picop, to enforce collection of the contested assessments; in
cited case was the donor's tax while the present suit pertains effect, the CIR denied Picop's protests.
to interest paid on the estate and inheritance tax. This
difference, however, submits no appreciable consequence to
the rationale of this Court's previous determination that PICOP appealed the assessments to CTA, which modified the
interests on taxes should be considered as interests on CIR’s findings and held PICOP liable for the reduced
indebtedness within the meaning of Section 30(b) (1) of the aggregate amount of P20,133,762.33.
Tax Code. The interpretation we have placed upon the said
section was predicated on the congressional intent, not on
the nature of the tax for which the interest was paid.
Picop and the CIR both went to the Supreme Court on
separate Petitions for Review of the above decision of the
PAPER INDUSTRIES CORPORATION OF THE PHILIPPINES CTA. Both petitions were referred by SC to the CA. Cases
(PICOP) vs. CA, CIR, and CTA (1995) were consolidated.

TtiIoOnNrSegistered with the Board of Investments ("BOI") as a preferred pioneer enterprise with respect to its integrated pulp and paper mill, and as a
CA: further reduced the liability of Picop.
[CHAPTER V: DEDUCTIONS] 19

in connection with the carrying on of the business of the


taxpayer. In the instant case, the CIR does not dispute that
Elevated the case again to SC, alleged: the interest payments were made by Picop on loans incurred
in connection with the carrying on of the registered
Picop: not liable at all to pay any of the assessments
operations of Picop, i.e., the financing of the purchase of
or any part thereof.
machinery and equipment actually used in the registered
CIR: Court of Appeals erred in finding Picop not liable operations of Picop. Neither does the CIR deny that such
for surcharge and interest on unpaid transaction tax interest payments were legally due and demandable under
and for documentary and science stamp taxes and in the terms of such loans, and in fact paid by Picop during the
allowing Picop to claim as deductible expenses and tax year 1977.
that Picop should be held liable for interest at 14%
The contention of CIR does not spring of the 1977 Tax
per annum for 3 years, and interest at 20% per annum
Code but from Revenue Regulations 2 Sec. 79. However, the
for a maximum of 3 years; and for a surcharge of 10%,
Court said that the term “interest” here should be construed
on Picop's deficiency income tax. Finally, the CIR
as the so-called "theoretical interest," that is to say, interest
contends that Picop is liable for the corporate
"calculated" or computed (and not incurred or paid) for the
development tax equivalent to five percent (5%) of its
purpose of determining the "opportunity cost" of investing
correct 1977 net income.
funds in a given business. Such "theoretical" or imputed
interest does not arise from a legally demandable interest-
bearing obligation incurred by the taxpayer who however
ISSUE: Whether Picop is entitled to deductions against wishes to find out, e.g., whether he would have been better
income of interest payments on loans for the purchase of off by lending out his funds and earning interest rather than
machinery and equipment. investing such funds in his business. One thing that Section 79
quoted above makes clear is that interest which does
HELD: constitute a charge arising under an interest-bearing
YES. Interest payments on loans incurred by a obligation is an allowable deduction from gross income.
taxpayer (whether BOI-registered or not) are allowed by the
NIRC as deductions against the taxpayer's gross income. The
CIR V LEDNICKY
basis is 1977 Tax Code Sec. 30 (b). Thus, the general rule is
that interest expenses are deductible against gross income
FACTS:
and this certainly includes interest paid under loans incurred

Taxation 1- SY 2015-2016 (Zarate)


Resp spouses V.E. Lednicky and Maria Valero Lednicky are 1959 sps filed amended return for 1957 claiming deductions
American Citizens residing in the Philippines and derived representing taxes paid to US Gov’t.
their income from Philippine sources for the taxable years in
* Tax court held that the taxes may be deducted because the
question.
Sps did not signify in their ITR a desire to avail themselves
1957 – Sps filed their ITR for 1956 reporting a gross income
of the benefits of paragraph 3(B) of Sec. 30
P1,017,287.65 and a net income of P733,809.44 on which
P317,395.40 was assessed after deducting P4,805.59 as
withholding tax. COMMON ISSUE: WON a citizen of the US residing in Phils
Sps paid 326,247.41 on April 1957 who derives income wholly from sources within the Phils may
March 1959 – Sps filed an amended ITR for 1956. They deduct from his gross income the income taxes he has paid to
claimed a deduction of P205,939.24 paid in 1956 to US gov’t. US gov’t for the taxable year?
Respondents requested refund of 112,437.90 HELD/RATIO:
CIR failed to answer the claim for refund, resps filed their • SC: CIR correct that the construction and wording of
petition with the Tax Court Sec. 30c(1)B of the Internal Revenue Act shows the
G.R. No. L-18169 formerly CTA case 570[different case/year] law’s intent that the right to deduct income taxes
is also a claim for refund in the amount of P150,269.00 as paid to foreign government from the taxpayer’s gross
alleged overpaid income tax for 1955 income is given only as an alternative or substitute to
Facts: his right to claim a tax credit for such foreign income
In Feb 1956 Sps filed ITR for 1955 = gross income of taxes
P1,7771,124.63 and net income of P1,052,550.67 o (B) – Income, war-profits, and excess profits
1956 – sps filed an amended ITR. Back in 1955, sps filed with taxes imposed by the authority of any foreign
the US Internal Revenue Agent in Manila their federal ITR for country; but this deduction shall be allowed in
the years 1947,1951-54 on income from Phil sources on a cash the case of a taxpayer who does not signify in
basis. 1958 – Sps amended their Phil ITR for 1955 to include his return his desire to have any extent the
the deductions of US Federal income taxes, interest accrued benefits of paragraph (3) of this subsection
up to May 15, 1955, and exchange and bank charges. CTA (relating to credit for foreign countries)
case 570 was filed. • So that unless the alien resident has a right to claim
2 0 C H
G. R . N o . such tax credit if he so chooses, he is precluded from
A P T E R V : D E D U C T IO NS
21 4 3 4 fo r m e rl y C T A Ca s e No. 783, facts deducting the foreign income taxes from his gross
are similar income.
but refer to Lednickys’ OTR for 1957 filed in Feb 1958. In
[CHAPTER V: DEDUCTIONS] 21

• For it is obvious that in prescribing that such contributing to the production of wealth that
deduction shall be allowed in the case of a taxpayer is being taxed
who does not signify in his return his desire to have o To allow an alien resident to deduct from his
any extent benefits of paragraph 3, the statute gross income whatever taxes he pays to his
assumes that the taxpayer in question may signify his own government amounts to conferring on the
desire to claim a tax credit and waive the deduction; latter the power to reduce the tax income of
otherwise, the foreign taxes would always be the Philippine government simply by
deductible and their mention in the list on non- increasing the tax rates on the alien resident.
deductible items in Sec. 30c might as well have been
omitted or at least expressly limited to taxes on
income from sources outside the Philippine Islands MARCELO STEEL CORPORATION
• Had the law intended that foreign income taxes could vs. COLLECTOR OF INTERNAL REVENUE
be deducted from gross income in any event,
regardless of the taxpayer’s right to claim a tax Facts: The petitioner is a corporation duly organized and
credit, it is the latter right that should be conditioned existing under and by virtue of the laws of the Philippines,
upon the taxpayer’s waiving the deduction with offices at Malabon, Rizal. It is engaged in three (3)
• No danger of double credit/taxation. industrial activities, namely, (1) manufacture of wire fence,
o Double taxation becomes obnoxious only (2) manufacture of nails, and (3) manufacture of steel bars,
where the taxpayer is taxed twice for the rods and other allied steel products. enjoined the benefits of
benefit of the same governmental entity the tax exemption under Republic Act No. 35.
o The Philippine government only receives the
proceeds of one tax On May 21, 1953, the petitioner filed an income tax return
o Justice and equity demand that the tax on the for the year 1952, reflecting a net income of P34,386.58
income should accrue to the benefit of the realized solely from its business of manufacturing wire fence,
Philippines an activity which is not tax exempt, and on March 31, 1954,
o Any relief from the alleged double taxation it filed its income tax return for the year 1953, showing a net
should come from the US since the former’s income of P58,329.00 realized from the same sources, i.e.,
right to burden the taxpayer is solely the manufacture of wire fence.
predicated in is citizenship, without

Taxation 1- SY 2015-2016 (Zarate)


On basis of the said income tax return filed by the petitioner
for the year 1952 and 1953 which did not reflect the financial
1) whether or not the petitioner may be allowed to deduct
of its tax exempt business activities, the respondent assessed
from the profits realized from its taxable business
the total sum of P12,750.
activities, the losses sustained by its tax except
industries, an
On October 1, 1954, the petitioner filed amended income tax
2) whether or not the action for refund, with regard to the
returns for taxable years 1952 and 1953, showing that bit
sum of P3,458.50 which was of the Tax Code.
suffered a net loss of P871,407.37 in 1952, and P10,956.29 in
1953. The said losses were arrived at by consolidating the
gross income and expenses and/or deductions of the Held: No.
petitioner in all its business activities. The fact that the petitioner is a corporate organized with a
single capital that answer for all its financial obligations
On October 1, 1954, the petitioner, claiming that instead of including those incurred in the tax exempt industries is of no
earning the net income shown in its original income tax moment. The intent of the law is to treat taxable or non-
returns for 1952 and 1953, it sustained the losses shown in its exempt industries as separate and distinct from new and
amended income tax returns for refund of the income taxes necessary industries which are tax-exempt for purposes of
for the said years amounting to P12,750.00 which it allegedly taxation. Section 7, Executive Order No. 341, series of 1950,
paid to the respondent. issued by the President of the Philippines pursuant to section
2, Republic Act No. 35, provides:
After more than ten months of waiting without any action
being taken by the respondent on the claim for refund, and Any industry granted tax exemption under the provisions of
in order to protect its right under Section 306 of the National Republic Act No. 35 shall report to the Secretary of Finance
Internal Revenue Code, the petitioner, on August 13, 1955, at the end of every fiscal rear a complete list and a correct
filed with this Court the instant petition for review. valuation of all real and personal property of its industrial
plant of factory: shall file a separate income tax return; shall
The Court of Tax Appeals held that "petitioner cannot deduct keep separetely the accounting records relative to the
from the profits realized from its taxable industries, the industry declared exempt; shall keep such records and
2 2
lo ss es submit such sworn statements as may be prescribed from
CsuHsAtaPinTeEdRbVy:itDs EtDaxUeCxTeImOpNtSbusiness time to time by the Secretary of Finance;1 (Emphasis
activities, . . ." supplied.)

Issue:
[CHAPTER V: DEDUCTIONS] 23

its exemption from the payment of income tax on its profits


And when Congress revised the provision of Republic Act No. derived from the operation of new and necessary industries,
35 by enacting onto law Republic Act No. 901 it incorporate the petitioner is exempt from the payment of other internal
similar provisions and provided that "Any industry granted tax revenue taxes directly payable by it, such as the fixed and
exemption under of Republic Act No. 35" shall "file a separate privilege tax on business, the percentage tax on the fixed
income tax return."2 and privilege tax on business, the percentage tax on the sales
of manufactured products, in respect to which exemption is
The petitioner states that it is not liable to pay income tax granted, the compensating tax on the articles, goods or
on its industries of manufacturing nails and steel bars, ros material exclusively used in the new and necessary industry,
and other allied steel product for the reason that it had and the documentary stamp tax (Exhibits 6 and 7). These
incurred loss in their operation and not because it is exempt exemption alone are enough to lighten its onerous financial
under the provision of Republic Act No. 35. It argues that by burden and reduce losses.
being allowed to deduct its gains derived from the operation
of its taxable or non-exempt industry of manufacturing wire The petitioner claims that unlike the United States Internal
fence, from the losses incurred in the operation of its tax- Revenue Code which expressly forbids the deduction of —
exempt industries of manufacturing nails and steels bars, rod
and other allied steel products, it would not receive the Any amount otherwise allowable as a deduction which is
benefit of double exemption under Republic Act No. 35 is to allocable to one or more classes of income other than
lighten the onerous financial burden and reduce the losses of interest (whether or not only any amount of income of that
the entrepeneur, yet it is not designed to assure him of a class or classes is received or accrued) wholly exempt from
return on his capital invested. the taxes imposed by this chapter [(Section 24 (a) (5)].

As already stated, the law intended to treat to treat taxable Our National Internal Revenue Code does not contain a
or non-excempt industry as separate and distinct from new similar prohibition. When in 1939 Commonwealth Act No.
and necessary industry, which is tax exempt, and did not 466, the National Internal Revenue Code, was enacted into
mean to grant an entrepreneur, engaged at the same time in law, the idea of granting tax exemption to new and necessary
a taxable or non-exempt industry and a new and necessary industries in the Philippines had not yet been thought of
industry, the benefit or privilege of deducting his gains or because there were no new and necessary industries being
profit derived from the operation of the first from the losses established or exploited. It was only in 1946, after the last
incurred in the operation of the second. Moreover, aside from World War, and after the Philippines became sovereign

Taxation 1- SY 2015-2016 (Zarate)


nation, that the establishment or exploitation of new and contractual obligation to produce and supply logs to the
necessary industries was stimulated. Hence the absence of a latter.
similar provision in out National Internal Revenue Code. This
absence, however, cannot be capitalized upon by the To afford itself adequate protection against loss or damage
petitioner in support of its theory. For, as already stated, on the performance bond, petitioner required San Jose and
when Congress enacted Republic Act No. 35 into law, it one Ramon Cuervo to execute an indemnity agreement
intended to segregate income derived from the operation of obligating themselves, solidarily, to indemnify petitioner for
new and necessary industries from that derived from the whatever liability it may incur by reason of said performance
operation of taxable or non-exempt industries. bond. Accordingly, San Jose constituted a chattel mortgage
on logging machineries and other movables in petitioner's
favor while Ramon Cuervo executed a real estate mortgage.
*Losses (Requisites for Deductibility)
San Jose later failed to deliver the logs to Galang Machinery
and the latter sued on the performance bond.

G.R. No. L-21520 December 11, 1967

PLARIDEL SURETY and INSURANCE COMPANY, petitioner, CFI: San Jose and petitioner = liable.

vs. CA and SC affirmed.

COMMISSIONER OF INTERNAL REVENUE, respondent.

In its income tax return for the year 1957, petitioner claimed
the amount of P44,490.00 (paid to Galang machinery) as
FACTS: deductible loss from its gross income and, accordingly, paid
the amount of P136.00 as its income tax for 1957. CIR
Petitioner Plaridel Surety & Insurance Co., is a domestic
disallowed deduction. Petitioner filed protest but was
corporation engaged in the bonding business. On November
denied.
9, 1950, petitioner, as surety, and Constancio San Jose, as
pr2in4cipCaHl,APsoTliEdRarVil:y DeExDeUcuCtTedIONa
Sperformance bond in the penal sum of P30,600.00 in favor
of the P. L. Galang Machinery Co., Inc., to secure the
performance of San Jose's
[CHAPTER V: DEDUCTIONS] 25

ISSUE: WON the amount of interest bond is an interest The alleged interest deduction not having been properly
deduction. litigated as an issue before the Tax Court, it is now too late
to raise and assert it before this Court.

HELD: NO.

Loss is deductible only in the taxable year it actually happens *Losses (Requisites for Deductibility)
or is sustained. However, if it is compensable by insurance or
G.R. No. L-18282 May 29, 1964
otherwise, deduction for the loss suffered is postponed to a
subsequent year, which, to be precise, is that year in which it COMMISSIONER OF INTERNAL REVENUE, petitioner,
appears that no compensation at all can be had, or that
there is a remaining or net loss, i.e., no full compensation. vs.

There is no question that the year in which the petitioner PRISCILA ESTATE, INC., and THE COURT OF APPEALS,
Insurance Co. effected payment to Galang Machinery respondents.
pursuant to a final decision occurred in 1957. However, under
the same court decision, San Jose and Cuervo were obligated FACTS:
to reimburse petitioner for whatever payments it would
The corporation duly filed its income tax returns for the
make to Galang Machinery. Clearly, petitioner's loss is
years 1949, 1950 and 1951. On 13 June 1952, however, it
compensable otherwise (than by insurance).itc-al It should
amended its income tax returns for 1951 and paid the tax
follow, then, that the loss deduction can not be claimed in
corresponding to the assessment made by the petitioner on
1957.
the basis of the returns, as amended; and on 13 September
The rule is that loss deduction will be denied if there is a 1952, the company claimed a refund of P4,941.00 as overpaid
measurable right to compensation for the loss, with ultimate income tax for the year 1950 for having deducted from gross
collection reasonably clear. So where there is reasonable income only the sum of P6,013.85 instead of P39,673.25 as
ground for reimbursement, the taxpayer must seek his its loss in the sale of a lot and building. Thereupon, the
redress and may not secure a loss deduction until he Commissioner of Internal Revenue conducted an investigation
establishes that no recovery may be had. of the company's income tax returns for 1949 through 1951
and, thereafter, granted a tax credit of P1,443.00 for 1950
but assessed on 3 November 1953 deficiency income taxes of
P3,575.49 for 1949 and P22,166.10 for 1951.

Taxation 1- SY 2015-2016 (Zarate)


The Priscila Estate, Inc., contested the deficiency
G.R. No. 125508 July 19, 2000
assessments and when the Commissioner of Internal Revenue
refused to reconsider them, the former brought suit to the CHINA BANKING CORPORATION,
tax court which after trial, rendered the decision that, in petitioner,
1961, the Commissioner elevated to this Supreme Court for vs.
review. COURT OF APPEALS, COMMISSIONER OF INTERNAL
REVENUE and COURT OF TAX APPEALS, respondents.
Topic: Ordinary Losses v Capital Losses
ISSUE: WON the cost of the barong-barong is an allowable
deduction. (The petitioner claims that the value of the Decisions:
demolished building should not be deducted from gross CTA: Denied in the inclusion for deductions valued
income but added to the cost of the building replacing it worthless securities.
because its demolition or removal was to make way for the
erection of another in its place.) 1äwphï1.ñët) CA: Affirmed CTA decision..

HELD: NO. SC: Petition denied. Affirmed CTA decision.

Tax court found that the removal of the "barong-barong", Facts:


instead of being voluntary, was forced upon the corporation 1. Sometime in 1980, petitioner China Banking Corporation
by the city engineer because the structure was a fire hazard; made a 53% equity investment in the First CBC Capital
that the rental income of the old building was about (Asia) Ltd., a Hongkong subsidiary engaged in financing and
P3,730.00 per month, and that the corporation had no funds investment with "deposit-taking" function. The investment
but had to borrow, in order to construct a new building. All amounted to P16,227,851.80, consisting of 106,000 shares
these facts, taken together, belie any intention on the part of with a par Value of P100 per share.
the corporation to demolish the old building merely for the
2. In 1986, it was shown that First CBC Capital (Asia), Ltd.,
purpose of erecting another in its place. Since the
has become insolvent. With the approval of Bangko
demolished building was not compensated for by insurance Sentral, petitioner wrote-off as being worthless its
or otherwise, its loss should be charged off as deduction investment in First CBC Capital (Asia), Ltd., in its 1987
fr2om6 gCroHsAsPinTcEoRmVe:. DEDUCTIONS Income Tax Return and treated it as a bad debt or as an
ordinary loss deductible from its gross income. This was
denied by the BIR stating among other things that they
should then be classified as "capital loss," and not as a bad
[CHAPTER V: DEDUCTIONS] 27

debt expense there being no indebtedness to speak of


between petitioner and its subsidiary.
G.R. Nos. 106949-50 December 1, 1995
Issue: WoN CA is correct in affirming CTA and BIR’s
decision that such worthless securities should be treated as PAPER INDUSTRIES CORPORATION OF THE PHILIPPINES
capital loss and not as bad debts? (PICOP), petitioner,
vs.
Basis of SC Decision: COURT OF APPEALS, COMMISSIONER OF INTERNAL
REVENUE and COURT OF TAX APPEALS, respondents.
1. No error on the CA ruling > In the case at bar, First CBC
Capital (Asia), Ltd., the investee corporation, is a G.R. Nos. 106984-85 December 1, 1995
subsidiary corporation of petitioner bank whose shares in
COMMISSIONER INTERNAL REVENUE, petitioner,
said investee corporation are not intended for purchase or
sale but as an investment. Unquestionably then, any loss vs.
therefrom would be a capital loss, not an ordinary loss, to PAPER INDUSTRIES CORPORATION OF THE PHILIPPINES,
the investor. Therefore, following Section 29(d)(4)(B) of THE COURT OF APPEALS and THE COURT OF TAX
the NIRC which states "(B) Securities becoming worthless. APPEALS, respondents..
- If securities as defined in Section 20 become worthless Facts:
during the tax" year and are capital assets, the loss
resulting therefrom shall, for the purposes of his Title, be Petitioner is registered with the BOI as a preferred pioneer
considered as a loss from the sale or exchange, on the last enterprise with respect to its integrated pulp and paper mill, and
day of such taxable year, of capital assets as a preferred non-pioneer enterprise with respect to its integrated
plywood and veneer mills. It received from the CIR two (2) letters
Note: In sum of assessment
(a) The equity investment in shares of stock held by CBC of and demand (a) one for deficiency transaction tax and for
approximately 53% in its Hongkong subsidiary, the First CBC
documentary and science stamp tax; and (b) the other for
Capital (Asia), Ltd., is not an indebtedness, and it is a
capital, not an ordinary, asset.91âwphi1 deficiency income tax for 1977, for an aggregate amount of
P88,763,255.00. Picop protested the assessment of deficiency
(b) Assuming that the equity investment of CBC has indeed
transaction tax and documentary and science stamp taxes. These
become "worthless," the loss sustained is a capital, not an
ordinary, loss.10 protests were not formally acted upon by respondent CIR. On 26
September 1984, the CIR issued a warrant of distraint on personal
(c) The capital loss sustained by CBC can only be deducted property and a warrant of levy on real property against Picop, to
from capital gains if any derived by it during the same
taxable year that the securities have become "worthless." enforce collection of the contested assessments; in effect, the CIR
denied Picop's protests. Thereupon, Picop went before the CTA.

Taxation 1- SY 2015-2016 (Zarate)


Picop and the CIR both went to the Supreme Court on separate registered with the BOI as a preferred pioneer enterprise — is that
Petitions for Review of the above decision of the CTA. In two (2) net operating losses cannot be carried over. Under our Tax Code,
Resolutions dated 7 February 1990 and 19 February 1990, both in 1977 and at present, losses may be deducted from gross
respectively, the Court referred the two (2) Petitions to the Court income only if such losses were actually sustained in the same year
of Appeals. The Court of Appeals consolidated the two (2) cases that they are deducted or charged off. Thus it is that R.A. No. 5186
and rendered a decision, dated 31 August 1992, which further introduced the carry-over of net operating
reduced the liability of Picop to P6,338,354.70. Picop now
maintains that it is not liable at all to pay any of the losses as a very special incentive to be granted only to registered
pioneer enterprises and only with respect to their registered
assessments or any part thereof. It assails the propriety of the operations. In the instant case, to allow the deduction claimed by
thirty-five percent (35%) deficiency transaction tax which the Court Picop would be to permit one corporation or enterprise, Picop, to
of Appeals held due from it in the amount of P3,578,543.51. Picop benefit from the operating
also questions the imposition by the Court of Appeals of the
deficiency income tax of P1,481,579.15, resulting from losses accumulated by another corporation or enterprise, RPPM. In
disallowance of certain claimed financial guarantee expenses and effect, to grant Picop's claimed deduction would be to permit Picop
claimed year-end adjustments of sales and cost of sales figures by to purchase a tax deduction and RPPM to peddle its accumulated
Picop's external auditors. 3 The CIR, upon the other hand, insists operating losses. We consider and so hold that there is nothing in
that the Court of Appeals erred in Section 7 (c) of
finding Picop not liable for surcharge and interest on unpaid R.A. No. 5186 which either requires or permits such a result.
transaction tax and for documentary and science stamp taxes and Indeed, that result makes non-sense of the legislative purpose
in allowing Picop to claim as deductible expenses. which may be seen clearly to be projected by Section 7 (c), R.A.
No. 5186. We conclude that the deduction claimed by Picop in the
ISSUE: Whether Picop is entitled to deduct against current
amount of P44,196,106.00 in its 1977 Income Tax Return must be
income net operating losses incurred by Rustan Pulp and Paper
disallowed.
Mills, Inc;.

HELD:

After prolonged consideration and analysis of this matter, the Court BAD DEBTS
is unable to agree with the CTA and Court of Appeals on the
G.R. No. L-22265 December 22, 1967
de d uct ib ili ty o f R PP M 's a cc u m u la t ed
2 8 against
l osses C H Picop's
A P 1977
T E gross
R Vincome.
: D EIt isDimportant
U C T to IO
noteN COLLECTOR OF INTERNAL REVENUE, petitioner,
S
at the outset that in our vs.
jurisdiction, the ordinary rule — that is, the rule applicable in GOODRICH INTERNATIONAL RUBBER CO., respondent.
respect of corporations not
[CHAPTER V: DEDUCTIONS] 29

TOPIC: Bad Debts respondent had not attached to its income tax returns a
statement showing the propriety of the deductions therein
FACTS: made for alleged bad debts.
Respondent was assessed by the CIR for Taxable Years Below are the accounts claimed by Goodrich as Bad Debts:
1951-1952. These assessments were based on disallowed
Disallowed by SC:
deductions, claimed by Goodrich, consisting of several
alleged bad debts, in the aggregate sum of P50,455.41, for Portillo's Auto Seat Cover (P730.00):
the year 1951, and the sum of P30,138.88, as representation
This debt was incurred in 1950. In 1951, the debtor paid
expenses allegedly incurred in the year 1952. Goodrich had P70.00, leaving a balance of P630.31. That same year, the
appealed from said assessments to the Court of Tax Appeals, account was written off as bad debt (Exhibit 3-C-4). Counsel
which, after appropriate proceedings, rendered, on June 8, for Goodrich had merely sent two (2) letters of demand in
1963, a decision allowing the deduction for bad debts, but 1951 (Exh. B-14). In 1952, the debtor paid the full balance
disallowing the alleged representation expenses, hence, this (Exhibit A).
appeal. Visayan Rapid Transit (P17,810.26):
This debt was, also, incurred in 1950. In 1951, it was charged
ISSUE: Whether or not the bad debts had been properly
off as bad debt, after the debtor had paid P275.21. No other
deducted for the year 1951. payment had been made.lawphil Taxpayer's Accountant
testified that, according to its branch manager in Cebu, he
RULING: Some but not all. The requirement of ascertainment had been unable to collect the balance. The debtor had
of worthlessness requires proof of two facts: (1) that the merely promised and kept on promising to pay. Taxpayer's
taxpayer did in fact ascertain the debt to be worthlessness, counsel stated that the debtor had gone out of business and
in the year for which the deduction is sought; and (2) that, in became insolvent, but no proof to this effect. was
so doing, he acted in good faith. Good faith on the part of introduced.
the taxpayer is not enough. He must show, also, that he had
reasonably investigated the relevant facts and had drawn a Bataan Auto Seat Cover (P373.13):
reasonable inference from the information thus obtained by This is the balance of a debt of P474.13 contracted in 1949.
him. The payments made, some in full, after some of the In 1951, the debtor paid P100.00. That same year, the
foregoing accounts had been characterized as bad debts, balance of P373.13 was charged off as bad debt. The next
merely stresses the undue haste with which the same had year, the debtor paid the additional sum of P50.00.
been written off. At any rate, respondent has not proven
that said debts were worthless. There is no evidence that the Tres Amigos Auto Supply (P1,370.31):
debtors can not pay them.lawphil.net It should be noted also
that, in violation of Revenue Regulations No. 2, Section 102,

Taxation 1- SY 2015-2016 (Zarate)


This account had been outstanding since 1949. Counsel for
The original account was P2,705.87, when, in 1950, it was
the taxpayer had merely sent demand letters (Exh. B-13)
turned over for collection to counsel for Goodrich (p. 156,
without success.
CTA Records). Counsel began sending letters of collection in
P. C. Teodoro (P650.00): April 1950. Interior Caltex made partial payments, so that as
In 1949, the account was P751.91. In 1951, the debtor paid of December, 1951, the balance outstanding was
P101.91, thus leaving a balance of P650.00, which the P1,505.87.lawphil.net The debtor paid P200, in 1952;
taxpayer charged off as bad debt in the same year. In 1952, P113.20, in 1954; P750.00, in 1961; and P300.00.00 in 1962.
the debtor made another payment of P150.00. The account had been written off as bad debt in 1951.
Ordinance Service, P.A. (P386.42):
In 1949, the outstanding account of this government agency Allowed by SC:
was P817.55. Goodrich's counsel sent demand letters (Exh.
B-8). In 1951, it paid Goodrich P431.13. The balance of San Juan Auto Supply (P4,530.64):
P386.42 was written off as bad debt that same year.
This account was contracted in 1950. Referred, for
Ordinance Service, P.C. (P796.26): collection, to respondent's counsel, the latter secured no
In 1950, the account was P796.26.lawphil It was referred to payment. In November, 1950, the corresponding suit for
counsel for collection. In 1951, the account was written off collection was filed (Exh. C). The debtor's counsel was
as a debt. In 1952, the debtor paid it in full. allowed to withdraw, as such, the debtor having failed to
meet him. In fact, the debtor did not appear at the hearing
National Land Settlement Administration (P3,020.76): of the case.lawphil.net Judgment was rendered in 1951 for
The outstanding account in 1949 was P7,041.51. Collection the creditor (Exh. C-2). The corresponding writ of execution
letters were sent (Exh. B-7). In 1951, the debtor paid (Exh. C-3) was returned unsatisfied, for no properties could
P4,020.75, leaving a balance of P3,020.76, which was written be attached or levied upon.
off, that same year, as a bad debt. This office was under
liquidation, and its Board of Liquidators promised to pay PACSA (P45.36),
when funds shall become available.
Philippine Naval Patrol (P14.18),
National Coconut Corporation (P644.74):
Surplus Property Commission (P277.68),
This account had been outstanding since 1949. Collection
letters were sent (Exh. B-12) without success. It was written Alvarez Auto Supply (P285.62):
off as bad debt in 1951, while the corporation was under a
Bo3a0rd These four (4) accounts were 2 or 3 years old in 1951. After
CofHLAiqPuTiEdaRtoVr:s,DwEhDiUchCTprIOomNiSsed to the collectors of the creditor had failed to collect the same,
pay upon availability its counsel wrote letters of demand (Exhs. B-10, B-11, B-6
of funds. In 1961, the debt was fully paid. and B-2) to no avail. Considering the small amounts involved
Interior Caltex Service Station (P1,505.87):
[CHAPTER V: DEDUCTIONS] 31

in these accounts, the taxpayer was justified in feeling that debts by PRC. PRC duly protested the assessment claiming that
the unsuccessful efforts therefore exerted to collect the
under the law, bad debts and interest expense are allowable
same sufficed to warrant their being written off.3
deductions.
Lion Shoe Store (P11,686.93),
When the BIR subsequently garnished some of PRC’s properties, the
Ruiz Highway Transit (P2,350.00), latter considered the protest as being denied and filed an appeal to
and the CTA which set aside the disallowance of the interest expense
and modified the disallowance of the bad debts by allowing 3
Esquire Auto Seat Cover (P3,536.94): accounts to be claimed as deductions. However, 13 supposed “bad
These three (3) accounts were among those referred to debts” were disallowed as the CTA claimed that these were not
counsel for Goodrich for collection. Up to 1951, when they substantiated and did not satisfy the jurisprudential requirement of
were written off, counsel had sent 17 Letters of demand to “worthlessness of a debt” The CA denied the petition for review.
Lion Shoe Store (Exh. B); 16 demand letters to Ruiz Highway
Transit (Exh. B-1); and 6 letters of demand to Esquire Auto ISSUE: Whether or not the CA was correct in disallowing the 13
Seat Cover (Exit. B-5) In 1951, Lion Shoe Store, Ruiz Highway accounts as bad debts.
Transit, and Esquire Auto Seat Cover had made partial
payments in the sums of P1,050.00, P400.00, and P300.00 RULING:YES.
respectively. Subsequent to the write-off, additional small
Both the CTA and CA relied on the case of Collector vs. Goodrich
payments were made and accounted for as income of
International, which laid down the requisites for “worthlessness of
Goodrich. Counsel interviewed the debtors, investigated
their ability to pay and threatened law suits. He found that a debt” to wit:
the debtors were in strained financial condition and had no
In said case, we held that for debts to be considered as "worthless," and
attachable or leviable property. Moreover, Lion Shoe Store thereby qualify as "bad debts" making them deductible, the taxpayer should
was burned twice, in 1948 and 1949. Thereafter, it continued show that (1) there is a valid and subsisting debt. (2) the debt must be
to do business on limited scale. Later; it went out of actually ascertained to be worthless and uncollectible during the
business. Ruiz Highway Transit, had more debts than assets. taxable year; (3) the debt must be charged off during the taxable year;
Counsel, therefore, advised respondent to write off these and (4) the debt must arise from the business or trade of the taxpayer.
accounts as bad debts without going to court, for it would be Additionally, before a debt can be considered worthless, the taxpayer
"foolish to spend good money after bad." must also show that it is indeed uncollectible even in the future.
Furthermore, there are steps outlined to be undertaken by the taxpayer to
PHILIPPINE REFINING CO ( UNILEVER COMPANY) v CA
prove that he exerted diligent efforts to collect the debts, viz.: (1) sending of
FACTS:Philippine Refining Corp (PRC) was assessed deficiency tax statement of accounts; (2) sending of collection letters; (3) giving the
payments for the year 1985 in the amount of around 1.8M. This account to a lawyer for collection; and (4) filing a collection case in
court.
figure was computed based on the disallowance of the claim of bad

Taxation 1- SY 2015-2016 (Zarate)


PRC only used the testimony of its accountant Ms. Masagana in order to
prove that these accounts were bad debts. This was considered by all 3 Upon investigation and examination of taxpayer's books and
courts to be self-serving. The SC said that PRC failed to exercise due papers, the CIR found that the reappraised assets
diligence in order to ascertain that these debts were uncollectible. In fact, depreciated in 1953 were the same ones upon which
PRC did not even show the demand letters they allegedly gave to some of depreciation was claimed in 1952. And for the year 1952, the
their debtors.
Commissioner had already determined, with taxpayer's
concurrence, the depreciation allowable on said assets to be
BASILAN ESTATES INC. vs. CIR and CTA (depreciation) P36,842.04, computed on their acquisition cost at rates fixed
by the taxpayer. Hence, the Commissioner pegged the
FACTS: deductible depreciation for 1953 on the same old assets at
P36,842.04 and disallowed the excess thereof in the amount
CIR, per examiners' report of February 19, 1959, assessed of P10,500.49.
Basilan Estates, Inc., a deficiency income tax of P3,912 for
1953 and P86,876.85 as 25% surtax on unreasonably ISSUE: W/N depreciation shall be determined on the
accumulated profits as of 1953 pursuant to Section 25 of the acquisition cost or on the reappraised value of the assets.
Tax Code. Basilan Estates, Inc. filed before the CTA a petition
for review of the Commissioner's assessment, alleging HELD: It shall be determined on the acquisition cost.
prescription of the period for assessment and collection;
Depreciation is the gradual diminution in the useful value of
error in disallowing claimed depreciations, travelling and
tangible property resulting from wear and tear and normal
miscellaneous expenses; and error in finding the existence of
obsolescense. The term is also applied to amortization of the
unreasonably accumulated profits and the imposition of 25%
value of intangible assets, the use of which in the trade or
surtax thereon.
business is definitely limited in duration. Depreciation
BEI claimed deductions for the depreciation of its assets up commences with the acquisition of the property and its
to 1949 on the basis of their acquisition cost. As of January owner is not bound to see his property gradually waste,
1, 1950 it changed the depreciable value of said assets by without making provision out of earnings for its replacement.
increasing it to conform with the increase in cost for their
The income tax law does not authorize the depreciation of an
replacement. Accordingly, from 1950 to 1953 it deducted
asset beyond its acquisition cost. Hence, a deduction over
from gross income the value of depreciation computed on the
and above such cost cannot be claimed and allowed. The
re3a2pprCaHiseAdPTvaElRueV. : DEDUCTIONS
reason is that deductions from gross income are
privileges, not matters of right. They are not created by
implication but upon clear expression in the law.
[CHAPTER V: DEDUCTIONS] 33

Moreover, the recovery, free of income tax, of an amount Mariano Zamora and his deceased sister Felicidad Zamora, bought a
more than the invested capital in an asset will transgress the piece of land located in Manila on May 16, 1944, for P132,000.00
underlying purpose of a depreciation allowance. For then and sold it for P75,000.00 on March 5, 1951. They also purchased a
what the taxpayer would recover will be, not only the lot located in Quezon City for P68,959.00 on January 19, 1944,
acquisition cost, but also some profit. Recovery in due time which they sold for P94,000 on February 9, 1951. The CTA ordered
thru depreciation of investment made is the philosophy the estate of the late Felicidad Zamora (represented by Esperanza
behind depreciation allowance; the idea of profit on the A. Zamora, as special administratrix of her estate), to pay the sum
investment made has never been the underlying reason for of P235.50, representing alleged deficiency income tax and
surcharge due from said estate.
the allowance of a deduction for depreciation.

Accordingly, the claim for depreciation beyond P36,842.04 or


in the amount of P10,500.49 has no justification in the law. Petitioner Mariano Zamora alleges that the CTA erred in disallowing
The determination, therefore, of the Commissioner of 3-½% per annum as the rate of depreciation of the Bay View Hotel
Internal Revenue disallowing said amount, affirmed by the Building but only 2-½%. In justifying depreciation deduction of 3-½
%, Mariano Zamora contends that (1) the Ermita District, where the
Court of Tax Appeals, is sustained. Bay View Hotel is located, is now becoming a commercial district;
(2) the hotel has no room for improvement; and (3) the changing
modes in architecture, styles of furniture and decorative designs,
ZAMORA v CIR "must meet the taste of a fickle public".

FACTS:
ISSUE: w/n the depreciation rate 2-1/2% is correct? YES

Mariano Zamora, owner of the Bay View Hotel and Farmacia


Zamora, filed his income tax returns. The CIR found that he failed
HELD:
to file his return of the capital gains derived from the sale of
certain real properties and claimed deductions which were not Section 30, of the Tax Code, provides that in computing net
allowable. The collector required him to pay deficiency income income, there shall be allowed as deductions all the ordinary and
tax. On appeal by Zamora, the CTA reduced the amount of necessary expenses paid or incurred during the taxable year, in
deficiency income tax. carrying on any trade or business. Since promotion expenses
constitute one of the deductions in conducting a business, same
must satisfy these requirements. Claim for the deduction of
Cases Nos. L-15289 and L-15281 promotion expenses or entertainment expenses must also be

Taxation 1- SY 2015-2016 (Zarate)


substantiated or supported by record showing in detail the amount legislative history of the statutes involved (Director of Lands v.
and nature of the expenses incurred. Abaya, et al., 63 Phil. 559). Zamora also contends that his basis for
applying the 3-½% rate is the testimony of its witness Mariano
Katipunan, who cited a book entitled "Hotel Management —
It is a fact, however, that the CTA, in estimating the reasonable Principles and Practice" by Lucius Boomer, President, Hotel Waldorf
rate of depreciation allowance for hotels made of concrete and Astoria Corporation. As well commented by the Solicitor General,
steel at 2-½%, the three factors just mentioned had been taken "while the petitioner would deny us the right to use Bulletin F, he
into account already. Said the CTA— would insist on using as authority, a book in Hotel management
Normally, an average hotel building is estimated to have a useful written by a man who knew more about hotels than about taxation.
life of 50 years, but inasmuch as the useful life of the building for
All that the witness did (Katipunan) . . . is to read excerpts from
business purposes depends to a large extent on the suitability of
the structure to its use and location, its architectural quality, the the said book (t.s.n. pp. 99-101), which admittedly were based on
rate of change in population, the shifting of land values, as well as the decision of the U.S. Tax Courts, made in 1928 (t.s.n. p. 106)".
the extent and maintenance and rehabilitation. It is allowed a The 2-½% rate of depreciation of the Bay View Hotel building, is
depreciation rate of 2-½% corresponding to a normal useful life of
approximately correct.
only 40 years (1955 PH Federal Taxes, Par 14 160-K). Consequently,
the stand of the petitioners can not be sustained.

As the lower court based its findings on Bulletin F, petitioner US vs LUDLEY


Zamora, argues that the same should have been first proved as a FACTS:
law, to be subject to judicial notice. Bulletin F, is a publication of
▪ Ludey brought this suit in the Court of Claims to recover an amount
the US Federal Internal Revenue Service, which was made after a exacted as additional taxes for 1917. The tax was assessed on the alleged
study of the lives of the properties. In the words of the lower gain from a sale in 1917 of oil mining properties which had been owned
court: "It contains the list of depreciable assets, the estimated and operated by him for several years. The Commissioner of Internal
average useful lives thereof and the rates of depreciation allowable Revenue determined that there was a gain on the sale of $26,904.15.
for each kind of property. (See 1955 PH Federal Taxes, Par. 14, 160 Ludey insists that there was a loss of $14,777.33. The amount sued for is
to Par. 14, 163-0) the tax assessed on the difference.
▪ The aggregate original cost of the properties was $95,977.33. Of this
amount, $30,977.33 was the cost of th equipment used in the business;
$65,000 the cost of the oil reserves. The 1917 sale price was $81,200.
It is true that Bulletin F has no binding force, but it has a strong
▪ CIR: Gain/Loss computed after deduction the depreciation/depletion
pe3rs4uasCivHeAePffTeEctRcVon:sDidEerDinUgCtThaIOt expenses.– the Commissioner deducted from the original cost $10,465.16
NthSe same has been the result of scientific studies and on account of depreciation of the equipment through wear and tear, and
observation for a long period in the United States after whose $32,258.81 on account of depletion of the reserves through the taking out
Income Tax Law ours is patterned." Verily, courts are permitted to of oil by the plaintiff, after March 1, 1913. In operating the properties,
look into and investigate the antecedents or the
[CHAPTER V: DEDUCTIONS] 35

Ludey disposed, in the form of oil, of part of his capital assets; that, in the
extraction of the oil, he consumed so much of the equipment as was was the gain or the loss depends primarily upon whether
represented by the depreciation, and disposed of so much of the oil deductions for depletion and depreciation are to be made
reserves as was represented by the depletion; that the sale of the from the original cost in determining gain or loss on sale of
properties made by him in 1917 was not a sale of all of the property oil mining properties. The question is one of statutory
represented by the original cost of $95,977.33, since physical equipment
to the amount of the depreciation and oil reserves to the amount of the construction or application. The Court of Claims entered
depletion had been taken from it during the preceding years, and that, for judgment for the plaintiff.
this reason, the cost to plaintiff of the net property sold in 1917 was not
$95,977.33, but $53,258.36.
▪ COURT OF CLAIMS: No deduction should be made – It held that no
deduction from original cost should be made here, because of the nature ISSUE: Whether or not depreciation of the mining e-uipment
of oil mining properties. It held that the depreciation was not deductible, should be deducted on the cost of the same to determine
because wear and tear of equipment was an expense or incident of the whether there was a gain or loss on the sale of the said
business.
equipment.

US vs. LUDLEY
RULING: YES
DOCTRINE
Until 1924, none of the revenue Acts provided in terms that
Depreciation should be taken into account in determining
in computing the gain from a sale of any property' a
whether there is a loss or gain in sale of properties for
deduction shall be made from the original cost on account of
purposes of income taxation.
depreciation and depletion during the period of operation.
FACTS But ever since March 1,1913, the revenue acts have required
that gains from sales made within the tax year shall be
Ludey brought this suit in the Court of Claims to recover an included in the taxable income of the year' and that losses on
amount exacted as additional taxes for 1917. The tax was sales may be deducted from gross income. We are of opinion
assessed on the alleged gain from a sale in 1917 of oil mining that the revenue acts should be construed as requiring
properties which had been owned and operated by him for deductions for both depreciation and depletion when
several years. The Commissioner of Internal Revenue determining the original cost of oil properties sold. The
determined that there was a gain on the sale of $26, 904.15. depreciation charge permitted as a deduction from the gross
Ludey insists that there was a loss of $14,777.33. The amount income in determining the taxable income of a business for
sued for is the tax assessed on the difference. ,hether there any year represents the reduction during the year' of the

Taxation 1- SY 2015-2016 (Zarate)


capital assets through wear and tear of the plant used. The a. Agricultural lands with a total area of 19,000 hectares in
amount of the allowance for depreciation is the sum which Nasugbu, Batangas
should be set aside for the taxable year' in order that at the
Tenants who have been tilling the lands expressed their desire to
end of the useful life of the plant in the business the
purchase from Roxas y Cia, the parcels which they actually occupied
aggregate of the sums set aside will (with the salvage value)
The govt, in line with the constitutional mandate to acquire big
suffice to provide an amount equal to the original cost. The
landed estates and apportion them among landless tenants-farmers,
theory underlying this allowance for depreciation is that by
persuaded the Roxas brothers to part with their landholdings
using up the plant a gradual sale is made of it. The
depreciation charged is the measure of the cost of the part The brothers agreed to sell 13,500 hec to the govt for P2.079Mn,
which has been sold. When the plant is disposed of after plus 300K survey and subdivision expenses
years of use' the thing then sold is not the whole thing Unfortunately, the govt did not have funds
originally acquired. The amount of the depreciation must be A special arrangement was made with the Rehabilitation Finance
deducted from the original cost of the whole in order to Corporation to advance to Roxas y Cia the amount of P1.5Mn as
determine the cost of that disposed of in the final sale of loan
properties. Any other construction would permit a double Under the arrangement, Roxas y Cia. allowed the farmers to buy the
deduction for the loss of the same capital assets. lands for the same price but by installment, and contracted with the
RFC to pay its loan from the proceeds of the yearly amortizations
DISPOSITIVE paid by the farmers
In 1953 and 1955, Roxas y Cia. derived from said installment
The decision of the Court of Claims is reversed and the case
payments a net gain of P42,480.83 and P29,500.71. 50% of said net
is remanded for further proceeding to determine the right
amount of depreciation. gain was reported for income tax purposes as gain on the sale of
capital asset held for more than one year pursuant to Sec. 34 of the
Tax Code
Roxas vs. CTA
GR No. L-25043 | April 26, 1968
b. Residential house and lot at Wright St., Malate, Manila
Facts: After the marriage of Antonio and Eduardo, Jose lived in the house
· 36 where he paid rentals of 8K/year to Roxas y Cia
CH DAoPnTPEeRdrVo:RDoExaDsUaCnTd IDOoNnSa
Carmen Ayala, both Spanish,
transmitted to their grandchildren by hereditary succession the
c. Shares of stocks in different corporations
following properties:
[CHAPTER V: DEDUCTIONS] 37

To manage the properties, Antonio Roxas, Eduardo Roxas and Jose Real estate dealer: any person engaged in the business of buying,
Roxas, the children, formed a partnership called Roxas y Compania selling, exchanging, leasing or renting property on his own account
On 1958, CIR demanded from Roxas y Cia the payment of real as principal and holding himself out as a full or part-time dealer in
estate dealer's tax for 1952 amtg to P150.00 plus P10.00 compromise real estate or as an owner of rental property or properties rented or
penalty for late payment, and P150.00 tax for dealers of securities offered to rent for an aggregate amount of three thousand pesos or
plus P10.00 compromise penalty for late payment. more a year:
Basis: house rentals received from Jose, pursuant to Art. 194 of the Section 194 of the Tax Code, in considering as real estate dealers
Tax Code stating that an owner of a real estate who derives a yearly owners of real estate receiving rentals of at least P3,000.00 a year,
rental income therefrom in the amount of P3,000.00 or more is does not provide any qualification as to the persons paying the
considered a real estate dealer and is liable to pay the corresponding rentals
fixed tax The fact that there were hundreds of vendees and them being paid for
The Commissioner further assessed deficiency income taxes against their respective holdings in installment for a period of ten years, it
the brothers for 1953 and 1955, resulting from the inclusion as would nevertheless not make the vendor Roxas y Cia. a real estate
income of Roxas y Cia of the unreported 50% of the net profits dealer during the 10-year amortization period
derived from the sale of the Nasugbu farm lands to the tenants, and The sale of the Nasugbu farm lands to the very farmers who tilled
the disallowance of deductions from gross income of various them for generations was not only in consonance with, but more in
business expenses and contributions claimed by Roxas y Cia and the obedience to the request and pursuant to the policy of our
Roxas brothers Government to allocate lands to the landless
The brothers protested the assessment but was denied, thus appealing It was the duty of the Government to pay the agreed compensation
to the CTA after it had persuaded Roxas y Cia. to sell its haciendas, and to
· CTA decision: sustained the assessment except the demand subsequently subdivide them among the farmers at very reasonable
for the payment of the fixed tax on dealer of securities and the terms and prices. But due to the lack of funds, Roxas y Cia.
disallowance of the deductions for contributions to the Philippine shouldered the Government's burden, went out of its way and sold
Air Force Chapel and Hijas de Jesus' Retiro de Manresa lands directly to the farmers in the same way and under the same
terms as would have been the case had the Government done it itself.
Issue: Should Roxas y Cia be considered a real estate dealer because The power of taxation is sometimes called also the power to destroy.
it engaged in the business of selling real estate Therefore it should be exercised with caution to minimize injury to
the proprietary rights of a taxpayer. It must be exercised fairly,
Ruling: NO, being an isolated transaction equally and uniformly

Taxation 1- SY 2015-2016 (Zarate)


Therefore, Roxas y Cia. cannot be considered a real estate dealer for · There is no question that the members of this group of citizens
the sale in question. Hence, pursuant to Section 34 of the Tax Code
do not receive profits, for all the funds they raised were for Manila's
the lands sold to the farmers are capital assets, and the gain derived
neediest families. Such a group of citizens may be classified as an
from the sale thereof is capital gain, taxable only to the extent of
association organized exclusively for charitable purposes mentioned
50%
in Section 30(h) of the Tax Code
d. Contribution to Our Lady of Fatima chapel at the FEU
As to the deductions
· University gives dividends to its stockholders
a. P40 tickets to a banquet given in honor of Sergio Osmena and
· Located within the premises of the university, the chapel in
P28 San Miguel beer given as gifts to various persons –
question has not been shown to belong to the Catholic Church or any
representation expenses
religious organization
· Representation expenses: deductible from gross income as
· The contributions belongs to the Far Eastern University,
expenditures incurred in carrying on a trade or business
contributions to which are not deductible under Section 30(h) of the
· In this case, the evidence does not show such link between the
Tax Code for the reason that the net income of said university injures
expenses and the business of Roxas y Cia
to the benefit of its stockholders
b. Contributions to the Pasay police and fire department and
other police departments as Christmas funds
No deficiency income tax is due for 1953 from Antonio Roxas,
· Contributions to the Christmas funds are not deductible for the
Eduardo Roxas and Jose Roxas. For 1955 they are liable to pay
reason that the Christmas funds were not spent for public purposes
deficiency income tax in the sum of P109.00, P91.00 and P49.00,
but as Christmas gifts to the families of the members of said entities
respectively
· Under Section 39(h), a contribution to a government entity is
deductible when used exclusively for public purposes
· As to the contribution to the Manila Police trust fund, such is *Items not Deductible
an allowable deduction for said trust fund belongs to the Manila
Police, a government entity, intended to be used exclusively for its G.R. No. L-26911 January 27, 1981
public functions.
ATLAS CONSOLIDATED MINING & DEVELOPMENT
c. Contributions to the Philippines Herald's fund for Manila's CORPORATION, petitioner,
neediest families
· 38 TChHeAcPonTtEriRbuVti:onDsEwDeUreCnToItOmNaSde vs.
to the Philippines Herald but
to a group of civic spirited citizens organized by the Philippines COMMISSIONER OF INTERNAL REVENUE, respondent.
Herald solely for charitable purposes
[CHAPTER V: DEDUCTIONS] 39

G.R. No. L-26924 January 27, 1981 For the year 1958, the assessment of deficiency income tax
COMMISSIONER OF INTERNAL REVENUE, petitioner, of P761,789.12 covers the disallowance of items claimed by
Atlas as deductible from gross income.
vs.
On October 25, 1962, the Secretary of Finance ruled that the
ATLAS CONSOLIDATED MINING & DEVELOPMENT exemption provided in Republic Act 909 embraces all new
CORPORATION and COURT OF TAX APPEALS, respondents. mines and old mines whether gold or other minerals.
Commissioner recomputed Atlas deficiency income tax
liabilities in the light of the ruling of the Secretary of
Finance. On June 9, 1964, the Commissioner issued a revised
assessment entirely eliminating the assessment of
FACTS: P546,295.16 for the year 1957. The assessment for 1958 was
reduced from P215,493.96 to P39,646.82 from which Atlas
This tax case (CTA No. 1312) arose from the 1957 and 1958 appealed to the Court of Tax Appeals, assailing the
deficiency income tax assessments made by the disallowance of the following items claimed as deductible
Commissioner of Internal Revenue, where the Atlas from its gross income for 1958.
Consolidated Mining and Development Corporation, was
assessed P546,295.16 for 1957 and P215,493.96 for 1958
deficiency income taxes.
CTA: allowed the ff as deductions: Transfer agent's fee, U.S.
Atlas is a corporation engaged in the mining industry stock listing expenses, Provision for contingencies EXCEPT
registered under the laws of the Philippines. On August 20, Stockholders relation service fee and suit expenses.
1962, the Commissioner assessed against Atlas the sum of
P546,295.16 and P215,493.96 or a total of P761,789.12 as
deficiency income taxes for the years 1957 and 1958. For the ISSUE: WON the expenses paid for the services rendered by a
year 1957, it was the opinion of the Commissioner that Atlas public relations firm P.K MacKer & Co. labelled as
is not entitled to exemption from the income tax under stockholders relation service fee is an allowable deduction as
Section 4 of Republic Act 909 because same covers only gold business expense under Section 30 (a) (1) of the National
mines. Internal Revenue Code.

Taxation 1- SY 2015-2016 (Zarate)


HELD: NO. services carrying on the selling campaign in an effort to sell
The principle is recognized that when a taxpayer claims a Atlas' additional capital stock of P3,325,000 is not an ordinary
deduction, he must point to some specific provision of the expense in line with the decision of U.S. Board of Tax Appeals
statute in which that deduction is authorized and must be in the case of Harrisburg Hospital Inc. vs. Commissioner of
able to prove that he is entitled to the deduction which the Internal Revenue.
law allows. As previously adverted to, the law allowing
expenses as deduction from gross income for purposes of the
income tax is Section 30 (a) (1) of the National Internal As held in the case of Vera vs. Fernandez, 30 this Court
Revenue which allows a deduction of "all the ordinary and emphatically said that taxes are the lifeblood of the
necessary expenses paid or incurred during the taxable year Government and their prompt and certain availability are
in carrying on any trade or business." An item of expenditure, imperious need. Upon taxation depends the Government's
in order to be deductible under this section of the statute, ability to serve the people for whose benefit taxes are
must fall squarely within its language. collected. To safeguard such interest, neglect or omission of
government officials entrusted with the collection of taxes
should not be allowed to bring harm or detriment to the
To be deductible as a business expense, three conditions people, in the same manner as private persons may be made
are imposed, namely: (1) the expense must be ordinary to suffer individually on account of his own negligence, the
and necessary, (2) it must be paid or incurred within the presumption being that they take good care of their personal
taxable year, and (3) it must be paid or incurred in affair. This should not hold true to government officials with
carrying in a trade or business. In addition, not only must respect to matters not of their own personal concern. This is
the taxpayer meet the business test, he must substantially the philosophy behind the government's exception, as a
prove by evidence or records the deductions claimed general rule, from the operation of the principle of estoppel.
under the law, otherwise, the same will be disallowed. The
mere allegation of the taxpayer that an item of expense is
ordinary and necessary does not justify its deduction. G.R. No. L-13325 April 20, 1961
SANTIAGO GANCAYCO, petitioner,
vs.
40 CHAPTER V: DEDUCTIONS
SC sustained the ruling of the tax court that the expenditure THE COLLECTOR OF INTERNAL REVENUE, respondent..
of P25,523.14 paid to P.K. Macker & Co. as compensation for Facts:
[CHAPTER V: DEDUCTIONS] 41

Gancayco filed his Income tax Return (ITR) for 1949. Deduction for expenses may be allowed, however in this case,
Gancayco was not able to prove any expense as there were no
Ø CIR notified him that his liability is Php 9.793.62, which he paid
receipts or other proofs. CTA AFFIRMED
1950

Ø CIR after a year wrote to Gancayco saying that there was tax due
from him for a total of Php 29,554.05

Ø Gancayco asked for reconsideration and the tax assessed


wasreduced

Ø CIR issued a warrant of distraint for the deficient liability

Ø Gancayco filed petition with CTA

CTA: Required Gancayco to pay Php 16, 860.31 for tax deficiency
in1949

Gancayco: the right to collect the deficiency income tax is barred


by the statute of limitations.

The 5 yr period for judicial action should be counted from May 12


50, the date of original assessment SC: Section 316 provides: The
civil remedies for the collection of internal revenue taxes, fees, or
charges, and any increment thereto resulting from delinquency
shall be (a) by distraint of goods, chattels, or effects, and other
personal property of whatever character, including stocks and other
securities, debts, credits, bank accounts, and interest in and rights
to personal property, and by levy upon real property; and (b) by
judicial action. Either of these remedies or both simultaneously
may be pursued in the discretion of the authorities charged with
the collection of such taxes. No exemption shall be allowed against
the internal revenue taxes in any case.

Taxation 1- SY 2015-2016 (Zarate)

You might also like